Esercizi sulla verifica dei limiti 4

Verifica del limite in funzioni

Home » Esercizi sulla verifica dei limiti 4

Presentiamo nel seguito i richiami di teoria utili allo svolgimento dell’esercizio. Per richiami più completi si veda la dispensa richiami di teoria.

Definizione 1 (limiti di funzioni) 
Sia A \subseteq \mathbb{R}, sia x_0 \in \overline{\mathbb{R}} un punto di accumulazione per A, sia f \colon A \to \mathbb{R} una funzione e sia \ell \in \overline{\mathbb{R}}. Si dice che \ell è il limite di f per x che tende a x_0 se, per ogni intorno V di \ell, esiste un intorno U di x_0 tale che

(1)   \begin{equation*} f(x) \in V \qquad \forall x \in U \cap A \setminus \{x_0\}. \end{equation*}

In tal caso si scrive

(2)   \begin{equation*} \lim_{x \to x_0} f(x) = \ell. \end{equation*}

Risulta utile vedere come si scrive esplicitamente la definizione 1 suddividendo le casistiche in cui x_0 \in \mathbb{R}, x_0 = -\infty, x_0=+\infty e \ell \in \mathbb{R}, \ell = -\infty, \ell=+\infty.

Definizione 2 (limiti destri e sinistri) ;
Siano A\subseteq \mathbb{R}, sia x_0 \in {\mathbb{R}} un punto di accumulazione sinistro per A, sia f \colon A \to \mathbb{R} una funzione e sia \ell \in \overline{\mathbb{R}}.
Si dice che \ell è il limite sinistro di f per x che tende a x_0 se, per ogni intorno V di \ell, esiste \delta>0 tale che

(3)   \begin{equation*} f(x) \in V \qquad \forall x \in (x_0-\delta,x_0) \cap A . \end{equation*}

In tal caso si scrive

(4)   \begin{equation*} \lim_{x \to x_0^-} f(x) = \ell. \end{equation*}

Analogamente si definisce il limite destro di f per x \to x_0 ed esso si indica con \displaystyle \lim_{x \to x_0^+} f(x).

 

Proposizione 1
Sia A \subseteq \mathbb{R}, sia x_0 un punto di accumulazione sinistro e destro per A, sia f \colon A \to \mathbb{R} e sia \ell \in \overline{\mathbb{R}}. Le seguenti affermazioni sono equivalenti:
\bullet \displaystyle \lim_{x \to x_0} f(x)= \ell;
\bullet \displaystyle \lim_{x \to x_0^+} f(x)= \lim_{x \to x_0^-} f(x) = \ell.

 

Testo dell’esercizio

Esercizio 4   (\bigstar\bigstar\largewhitestar\largewhitestar\largewhitestar).

Si verifichino, mediante la definizione, i seguenti limiti:

  1. \displaystyle \lim_{x \to 0^+} \dfrac{1}{x}=+\infty;
  2. \displaystyle \lim_{x \to +\infty} \dfrac{1}{x}=0;
  3. \displaystyle \lim_{x \to 2} \dfrac{1}{x}=\dfrac{1}{2}.

 

Svolgimento.

Risolviamo i diversi punti separatamente. L’esercizio riguarda lo studio dei limiti della funzione f \colon \mathbb{R} \setminus \{0\} \to \mathbb{R} definita da

(5)   \begin{equation*} f(x) = \frac{1}{x} \qquad \forall x \in \mathbb{R} \setminus \{0\}, \end{equation*}

il cui grafico è rappresentato in figura 4a. Osserviamo che ogni numero reale è un punto di accumulazione destro e sinistro del dominio \mathbb{R} \setminus \{0\} di f; inoltre anche +\infty è un punto di accumulazione del dominio di f, essendo quest’ultimo illimitato superiormente. Dunque i limiti richiesti dalla traccia sono significativi e possono essere studiati.

Figura 4a: la funzione f dell’esercizio 4.

 

  1. Occorre calcolare il limite destro in x_0=0 della funzione f. Poiché \ell=+\infty, siamo nel caso 3 riportato dalla tabella 1. Scegliamo dunque M \in \mathbb{R}. In virtù dell’osservazione 1 dei richiami di teoria, possiamo limitarci a considerare M>0. Dato che

    (6)   \begin{equation*} f(x) > M \iff \frac{1}{x} > M \iff 0< x < \frac{1}{M}, \end{equation*}

    scegliendo \delta=\frac{1}{M} otteniamo

    (7)   \begin{equation*} x \in (0,\delta) \implies f(x) > M, \end{equation*}

    cioè quanto volevamo provare. La situazione è rappresentata in figura 4b.

    Figura 4b: illustrazione del punto 1 dell’esercizio 4. Abbiamo fissato l’intorno (M,+\infty) di +\infty, rappresentato in rosso, e abbiamo mostrato che esiste un intorno destro di 0, definito da U=(0,\delta)=\left (0,\frac{1}{M} \right ), rappresentato in verde, con la seguente proprietà: se x \in U, allora f(x) \in (M,+\infty).

     

  2. Verifichiamo ora che \lim_{x \to +\infty} \frac{1}{x}=+\infty. A tal fine, dobbiamo dimostrare che vale la formula nel caso 7 della tabella 1. Fissiamo \varepsilon>0 e dunque un intorno (-\varepsilon,\varepsilon) del limite \ell=0 e mostriamo che esiste H \in \mathbb{R}, cioè un intorno (H,+\infty) di +\infty, con la seguente proprietà:

    (8)   \begin{equation*} x \in (H,+\infty) \implies |f(x) - 0| = |f(x)| < \varepsilon. \end{equation*}

    Osserviamo che

    (9)   \begin{equation*} |f(x)|< \varepsilon \iff \left | \frac{1}{x} \right | < \varepsilon \iff |x| > \frac{1}{\varepsilon}. \end{equation*}

    Dunque, se scegliamo H = \frac{1}{\varepsilon}, abbiamo

    (10)   \begin{equation*} x \in (H,+\infty) \implies |f(x)|< \varepsilon, \end{equation*}

    come desiderato. Il procedimento è illustrato in figura 4c.

    Figura 4c: illustrazione del punto 2 dell’esercizio 4. Fissato l’intorno (-\varepsilon,\varepsilon) di 0, rappresentato in rosso, l’intorno U=(H,+\infty)=\left (\frac{1}{\varepsilon},+\infty \right ) di 0 rappresentato in verde è tale che, se x \in U, allora f(x) \in (-\varepsilon,+\varepsilon).

     

  3. Dobbiamo verificare che valga la formula del primo caso nella tabella 1 con x_0=2 e \ell = \frac{1}{2}.
    Fissiamo \varepsilon>0 e, in virtù dell’osservazione 1 dei richiami di teoria, , possiamo limitarci a studiare il caso in cui \varepsilon \in \left (0, 1\right ). Osserviamo preliminarmente che

    (11)   \begin{equation*} \left | f(x) - \ell \right | = \left | \frac{1}{x} - \frac{1}{2} \right | = \left | \frac{2-x}{2x}\right | \leq \frac{1}{2}|2-x| \qquad \forall x \geq 1, \end{equation*}

    dove l’ultima uguaglianza segue appunto da x \geq 1.
    Poiché \varepsilon\in (0,1), da x \in (2-\varepsilon,2+\varepsilon) segue x \geq 1, quindi possiamo scegliere \delta=2\varepsilon e applicare (11) per ottenere

    (12)   \begin{equation*} x \in (2-2\varepsilon,2+2\varepsilon) \implies \left | f(x) - \ell \right | \leq \frac{1}{2}|2-x| < \frac{1}{2} 2\varepsilon = \varepsilon. \end{equation*}

Riguardo l’ultima richiesta dell’esercizio, analogamente al punto 1 si può mostrare che

(13)   \begin{equation*} \lim_{x \to 0^-} \frac{1}{x} = -\infty, \end{equation*}

evidente anche dalla figura 4a. Poiché i limiti sinistro e destro di f in 0 non coincidono, la proposizione 1 implica che \lim_{x \to 0} \frac{1}{x} non esiste.

error: Il contenuto è protetto!!